K
Khách

Hãy nhập câu hỏi của bạn vào đây, nếu là tài khoản VIP, bạn sẽ được ưu tiên trả lời.

23 tháng 11 2016

5/36

 

HQ
Hà Quang Minh
Giáo viên
1 tháng 10 2023

Số phần tử của không gian mẫu là \(n\left( \Omega  \right) = 36\).

Gọi E là biến cố tổng số chấm xuất hiện trên hai con xúc xắc bằng 4 hoặc bằng 6. Khi đó ta có \(E = \left\{ {\left( {1,3} \right);\left( {2,2} \right);\left( {3,1} \right);\left( {1,5} \right);\left( {2,4} \right);\left( {3,3} \right);\left( {4,2} \right);\left( {5,1} \right)} \right\} \Rightarrow n\left( E \right) = 8\).

Vậy xác suất của biến cố E là \(P\left( E \right) = \frac{{n\left( E \right)}}{{n\left( \Omega  \right)}} = \frac{8}{{36}} = \frac{2}{9}\).

7 tháng 9 2019

Đáp án A

5 tháng 11 2016

các trường hợp là :

3-1

4-2

5-3

6-4

=> xác suất P=4/36=1/9

HQ
Hà Quang Minh
Giáo viên
1 tháng 10 2023

Ta có số phần tử của không gian mẫu là \(n\left( \Omega  \right) = 36\).

a) Ta có \(E = \left\{ {\left( {1,1} \right);\left( {1,2} \right);\left( {2,1} \right);\left( {2,2} \right)} \right\}\). Suy ra \(n\left( E \right) = 4\) và \(P\left( E \right) = \frac{4}{{36}} = \frac{1}{9}\).

b) Ta có \(F = \{(1,5);(2,5);(3,5);(4,5);(5,5);(6,5);(1,6);(2,6);(3,6);(4,6);(5,6);(6;6)\}\). Suy ra \(n\left( F \right) = 12\). Vậy \(P\left( F \right) = \frac{{12}}{{36}} = \frac{1}{3}\).

c) Ta có \(G = \{ \left( {1;1} \right);\left( {1,2} \right);\left( {1,3} \right);\left( {1,4} \right);\left( {1,5} \right);\left( {2,1} \right);\left( {2,2} \right);\left( {3,1} \right);\left( {4,1} \right);\left( {5,1} \right)\} \). Suy ra \(n\left( G \right) = 10\). Vậy \(P\left( G \right) = \frac{{10}}{{36}} = \frac{5}{{18}}\).

d) Ta có \(H = \{ ( 1,1 );( 1,2 );( 2,1 );( 1,4 );( 2,3 );( 3,2 );( 4,1 );( 1,6 ) ;( 2,5 ) ;( 3,4 );( 4,3 );( 5,2 );( 6,1 );( 5,6 );( 6,5 ) \}\). Suy ra \(n\left( H \right) = 15\). Vậy \(P\left( H \right) = \frac{{15}}{{36}} = \frac{5}{{12}}\).

HQ
Hà Quang Minh
Giáo viên
1 tháng 10 2023

Số phần tử của không gian mẫu là \(n\left( \Omega  \right) = 36\)

Gọi E là biến cố \(E = \left\{ {\left( {1,1} \right);\left( {1;2} \right);\left( {1,3} \right);\left( {2  ;1} \right);\left( {2;2} \right);\left( {3,1} \right)} \right\}\) suy ra \(n\left( E \right) = 6\)

Vậy \(P\left( E \right) = \frac{6}{{36}} = \frac{1}{6}\).

Chọn B